2

I'm struggling to understand a confusing feature of the Born rule in quantum mechanics, which is that the probability of an outcome appears to depend on how coarsely grained our measurements are. Consider the following normalized wave function in the position basis:

$$|\psi\rangle = \frac{1}{\sqrt{2}}|x=+1\rangle + \frac{1}{\sqrt{2}}|x=-1\rangle$$

Here the Born rule tells us that the probability of measuring a particle at each position is $\frac{1}{2}$. But what if our technology improves such that our measurement apparatus is able to discern a fine splitting around x=-1, such that the wave function is actually:

$$|\psi\rangle = \frac{1}{\sqrt{2}} |x=+1\rangle + \frac{1}{\sqrt{2}} \left(\frac{1}{2}|x=-1.01\rangle + \frac{1}{2} |x=-0.99\rangle \right)$$

Here is a picture that may help clarify what I'm referring to: enter image description here

The problem (or at least what is confusing to me) is that by merely decomposing a lump of amplitude into two equal parts we have changed the experimental outcome. Where before the probability was $\frac{1}{2}$ to measure the particle on the LHS or RHS, after we normalize the above state, it becomes:

$$|\psi\rangle = \sqrt{\frac{2}{3}}|x=+1\rangle + \frac{1}{\sqrt{6}}|x=-1.01\rangle + \frac{1}{\sqrt{6}}|x=-0.99\rangle$$

In other words the probability of measuring the particle on the LHS has been changed to 1/3 and on the RHS is now 2/3. The confusing thing is that all I have done is draw a line of demarcation between two halves of an amplitude distribution; I wouldn't have thought I was changing the physical state, but apparently I am because we are forced to re-normalized the state, resulting in different course-grained experimental outcomes.

Am I doing something wrong, or is this just a weird feature of quantum mechanics? If it is just a weird feature of quantum mechanics, how is it that it is not actually inconsistent, given that I merely used formalism to re-label two halves of a lump of what is otherwise the same amplitude distribution?

EDIT Adding the picture below which hopefully more succinctly illustrates the confusion:

enter image description here

user1247
  • 7,288
  • 2
    What do you mean by "such that the wave function is actually"? What states you can distinguish has no effect on the actual state. Your state is $\lvert 1 \rangle + \lvert -1 \rangle$. What you are going to measure it with has no influence it, but what it will be projected on when applying the Born rule. If your apparatus can measure with a width of $0.1$, then that just means that you model the measurement by projecting on something like $\int f_{x_0}(x)\lvert x \rangle\mathrm{d}x$ instead of a single position eigenstate $\lvert x_0 \rangle$ where $f(x)$ has a width of $0.1$ around $x_0$. – ACuriousMind Jan 06 '16 at 17:33
  • @ACuriousMind, the actual state I have in mind is what is shown in the picture, not |1>+|-1>, which I understood to be an approximation if all our experiment could do was determine whether the electron (for example) went right or left (for example after a stern-gerlach), even though if we had a better measurement apparatus we would see a distribution determined by the wave function shown in the picture. Why is my question being downvoted? Can I improve it somehow? – user1247 Jan 06 '16 at 18:04
  • @ACuriousMind, if I model the measurement in the way you suggest, I don't see how the same question doesn't arise, the logical sequence just goes in the opposite direction of what is given in my post. I would start with the last state, and then project it onto |+1> and |-1> and still end up with the same confusion. – user1247 Jan 06 '16 at 18:07
  • Your picture doesn't make much sense to me - why do you "divide integrated amplitude in half"? And why would you project onto $\lvert +1\rangle+\lvert -1\rangle$? The measurement process is modeled by projecting onto the states the apparatus measures. – ACuriousMind Jan 06 '16 at 18:12
  • @ACuriousMind, my understanding is that if we look at a distribution of wave function amplitude in the position basis like shown at the top of the picture, it represents an integral over |x> states with weights as shown in the figure. That is the actual state: some distribution over position basis states. But our measurement apparatus only measures, say, |+1> or |-1>, because it lacks the resolution to see the functional form shown in the picture. The "divide integrated amplitude in half" is imagining our apparatus improving such that it can resolve the two sides of the LHS distribution... – user1247 Jan 06 '16 at 19:04
  • @ACuriousMind, ...which was previously mapped to |-1> due to experimental resolution. Since the actual state is an integral over position basis states, it seems correct that we can integrate the two sides of the LHS distribution separately, so that we have 1/2|-0.99> and 1/2|-1.01>. – user1247 Jan 06 '16 at 19:06

2 Answers2

6

The Born rule:

  1. Take the initial state vector of the subject.
  2. Take a measurement operator.
  3. Take the projection of the state vector into an eigenspace of the measurement operator.
  4. Find the squared norm of each vector (initial and projection).
  5. Take the ratio of the smaller over the larger.
  6. Interpret that as the frequency of getting the measurement result associated with that eigenspace of that measurement.
  7. When the device has had time to transition to a new device state, it registers a particular device result. It ends up in a potentially new state of the device, but the device state is associated with the eigenspace of the measurement.
  8. When the device has had time to transition to a new state of the device, then the state of the subject has become the projection of the original state into that eigenspace.
  9. The new joint state is a product of the device state and the subject state.

That's it. Lets look at a famous example you have probably studied before, the hydrogen atom, the part with the degrees of freedom associated with the relative coordinates of the electron relative to the center of mass. You can write some bound states like $|nlm\rangle,$ where the center of mass energy is $-13.6eV/n^2$ less than the ionization energy, the square of the orbital angular momentum is $\hbar^2l(l+1),$ and the $\hat z$ component of the orbital angular momentum is $\hbar m.$

Now if you measure the energy of the hydrogen (the subject) with a device you need to note that the state of the device and the state of the subject (the hydrogen) are different things.

So you might start out with a joint state like $$\left(\frac{1}{\sqrt 2}|100\rangle+\frac{1}{2}|211\rangle+\frac{1}{2}|210\rangle\right)\otimes |0\rangle.$$

Where the $|0\rangle$ is the state of the measurement device before the measurement. Then afterwards you end up with $|100\rangle\otimes |13.6eV\rangle$ 50% of the time and end up with $$\left(\frac{1}{\sqrt 2}|211\rangle+\frac{1}{\sqrt 2}|210\rangle\right)\otimes |13.6eV/4\rangle$$ the other 50% of the time.

Note that when the detector ended up in the state $|13.6eV\rangle$ there was only only possible state for the hydrogen, the hydrogen had to be in the ground state. But instead, when the detector is in the state $13.6eV/4\rangle$ and you didn't know the original state of the hydrogen, then you haven't learned whether the hydrogen is now in the state $|211\rangle,$ $|210\rangle,$ $|21-1\rangle$ or even $\alpha|211\rangle+\beta|210\rangle+\gamma|21-1\rangle.$

That 3d vector subspace of hydrogen states corresponding to the same energy is called an eigenspace of the energy operator. And when you measure the energy, you only find out $n$ you don't find out $l$ or $m.$ This is normal. And it's what Peter was calling the more general measurement, but you've probably seen it before.

This does mean that we have to be careful to use labels and names that distinguish between the way the state is, and the way the measurement comes out. The state needs to have enough information to fully specify any possible measurement. The detector could have a whole 2d, 3d or more subspace associated with it.

Now if there was a magnetic field those different $m$ states might technically have slightly different energy states. But the center of mass of the hydrogen could have some motion too. So when you try to measure the energy precisely you might not be able to tell the hydrogen moving towards your light source and the lower energy versus the hydrogen moving away from your light source and the higher energy. This means you might do a coarse measurement of energy.

If instead you cooled some hydrogen gas down enough so that the velocities were small and turned up the magnetic field strong enough then the spacing between the different $m$ states might be larger than the Doppler broadening. Now you might be able to detect whether something is in the state $|211\rangle,$ $|210\rangle,$ $|21-1\rangle$ or even $\alpha|211\rangle+\beta|210\rangle+\gamma|21-1\rangle.$

So these kinds of coarse or fine grained measurements are possible. But keep in mind that the possible states of the pointer, can in general allow a whole 2d or 3d or even more vector subspace of the subject.

So lets look at your example. It's actually 100% like the hydrogen example in the math, so make you understand that example, where every $\frac{1}{\sqrt 2}$ and $\frac{1}{2}$ came from and why.

So first we need a notation. I will use $|[a,b)\rangle $ for the state $\Psi_{[a,b)}$ that sends $x$ to $\Psi_{[a,b)}(x)=0$ if $x\notin[a,b)$ and sends $x$ to $\Psi_{[a,b)}(x)=\frac{1}{\sqrt{b-a}}$ if $x\in[a,b).$ Note that these states have unit norm. And we shall use the notation $x\mapsto\Psi_{[a,b)}(x)$ as another way to denote a function.

Next we define our operators. The coarse grained operator will be denoted $LR$ and we can define it by

$$LR:[\Psi=(x\mapsto\Psi(x))]\mapsto \left[LR\Psi=\left(x\mapsto[(LR\Psi)(x)=\frac{x}{|x|}\Psi(x)]\right)\right].$$

Note that this a perfectly well defined operator and in particular $\Psi_{[a,b)}$ is an eigenvector with eigenvalue $-1$ if $a\lt b\leq 0$ and $\Psi_{[a,b)}$ is an eigenvector with eigenvalue $+1$ if $0\leq a\lt b.$ And the size of the subspace is huge for each eigenvalue. This is a very very coarse operator. But it can separate things on the left from things on the right. And you can instead make finer operators such as $ABC$ if you want to resolve things better. We can define $ABC$ as follows:

$$ABC:[\Psi=(x\mapsto\Psi(x))]\mapsto \left[ABC\Psi=\left(x\mapsto[(ABC\Psi)(x)=\frac{x+1}{2|x+1|}\Psi(x)+\frac{x}{2|x|}\Psi(x)]\right)\right].$$

This is also a perfectly fine operator. It has eigenvalues $-1,$ $0,$ and $+1$ and in particular $\Psi_{[a,b)}$ is an eigenvector with eigenvalue $-1$ if $a\lt b\leq -1$ and $\Psi_{[a,b)}$ is an eigenvector with eigenvalue $+1$ if $0\leq a\lt b$ and $\Psi_{[a,b)}$ is an eigenvector with eigenvalue $0$ if $-1\leq a\lt b\leq 0.$

Clearly we could make operators that are even finer. But also keep in mind that each operator has a giant eigenspace associated with each eigenvalues, so just like the energy of the hydrogen atom we don't know the state if the subject just by stating the state of the measurement device. That's what is means to make a coarse measurement. And it's very common, it is called having a degenerate eigenvalue.

If you followed these steps you would have no questions, zero questions. So what issue are you having? You didn't do the above. Instead, you made a large number of very serious errors, including but not limited to:

  • You act as if your initial state is determined by your operator, it isn't.
  • You try to compute with some numbers instead of finding eigenspaces and doing projections. And you don't get your numbers correctly.
  • You look at areas under curves instead of areas under squares of curves.
  • You literally refuse to write the actual correct wavefunctions, operators and so forth.

If your subject state is originally $\Psi=\frac{1}{\sqrt 2}\Psi_{[-1.2,-0.8)}+\frac{1}{\sqrt 2}\Psi_{[0.8,1.2)}$ then please do verify that it is of unit norm, and that it sends $x$ to $\sqrt{5/4}$ when $x\in[-1.2,-0.8)\cup[0.8,1.2)$ and sends $x$ to zero otherwise. Then please verify that $\Psi_{[-1.2,-0.8)}=\frac{1}{\sqrt 2}\Psi_{[-1.2,-1.0)}+\frac{1}{\sqrt 2}\Psi_{[-1.0,-0.8)}.$

Now under the coarse operator $LR$ we get the final states of the detector are $|L\rangle$ and $|R\rangle.$ And under the fine operator $ABC$ we get the final states of the detector are $|A\rangle,$ $|B\rangle,$ and $|C\rangle.$

Thus if the initial state of the subject is $\Psi=\frac{1}{\sqrt 2}\Psi_{[-1.2,-0.8)}+\frac{1}{\sqrt 2}\Psi_{[0.8,1.2)}$ then since $\Psi_{[-1.2,-0.8)}=\frac{1}{\sqrt 2}\Psi_{[-1.2,-1.0)}+\frac{1}{\sqrt 2}\Psi_{[-1.0,-0.8)}$ we get

$$\Psi=\frac{1}{\sqrt 2}\Psi_{[-1.2,-0.8)}+\frac{1}{\sqrt 2}\Psi_{[0.8,1.2)}$$

and $$\Psi=\frac{1}{\sqrt 2}\left(\frac{1}{\sqrt 2}\Psi_{[-1.2,-1.0)}+\frac{1}{\sqrt 2}\Psi_{[-1.0,-0.8)}\right)+\frac{1}{\sqrt 2}\Psi_{[0.8,1.2)}$$

The first way of writing it, writes it entirely in terms of eigenvectors of $LR$ so you can see right away that if we measure it in terms of $LR$ we get it in state $\Psi_{[-1.2,-0.8)}\otimes|L\rangle$ 50% of the time and in state $\Psi_{[0.8,1.2)}\otimes|R\rangle$ 50% of the time.

But if we write it that second way it is terms of common eigenvector of both operators so we could see both measurements equally well. For coarse one sends it to $\left(\frac{1}{\sqrt 2}\Psi_{[-1.2,-1.0)}+\frac{1}{\sqrt 2}\Psi_{[-1.0,-0.8)}\right)\otimes|L\rangle$ 50% of the time and in state $\Psi_{[0.8,1.2)}\otimes|R\rangle$ 50% of the time.

And for the fine one it sends it to$\Psi_{[-1.2,-1.0)}\otimes|A\rangle$ 25% of the time, $\Psi_{[-1.0,-1.2)}\otimes|B\rangle$ 25% of the time, and to $\Psi_{[0.8,1.2)}\otimes|C\rangle$ 50% of the time.

This is all the physics. Make sure you understand the physics. Make you understand the math. And I tried to make it 100% look like your picture, except doing it correctly by having my notation always express things in terms of unit vectors to make the probabilities easy to compute. The wavefunction is zero or $\sqrt{5/4}$ every where before the measurement and it is different after the measurement

Timaeus
  • 25,523
  • I think you are begging the question, or at least that possibility is at the heart of my confusion. To elaborate, when you say "But a wave like ... is not even normalized," you seem to be ignoring that I stated explicitly in my question that indeed the wave is not normalized, which was the reason for the discussion that follows it, where I proceed to re-normalize the state, and am disturbed to find that the probabilities we then find are non-intuitive. Continued... – user1247 Jan 07 '16 at 01:57
  • When you say "And the size of the whole part needs to have a square of 1/2, like it did before you broke it into pieces," you are pointing out that I should have forced the normalization from the outset. But if you do that, you are no longer cleaving the amplitude in two, rather you are altering the total integrated amplitude function around x=-1 in order to keep the state normalized. In other words, we are both doing something to the state to make sure it is normalized. Continued... – user1247 Jan 07 '16 at 02:03
  • But I am keeping the relative amplitudes constant by merely re-labeling the two-halves of the lump on the LHS of the picture (and then re-normalizing the overall state), while you are re-weighting the relative amplitudes in order to make sure that the experimental outcome for the RHS still adds up to 50% (hence "begging the question"). Why should I do it your way and not my way? Where am I told that in the formalism of QM? (Sorry if I am sounding combative, I'm sure you're correct, but I don't yet understand at all why you are correct). – user1247 Jan 07 '16 at 02:06
  • Also, I don't understand the first question you pose. Where did the 5 and -5 come from? And why is the first wave function you write down not normalized (or is that a point you are making)? Your second equation I of course agree with, and the third also, hence the discussion in my question about the effect of properly normalizing the state. The central confusion I have is the fact that your third equation seems to follow from linearity and the def of <psi|x> as the integral (ie unweighted sum) over basis states, while the 4th is of course normalized but its relation to eq #2 begs the question. – user1247 Jan 07 '16 at 02:18
  • @user1247 Why did you write the second one instead of the first one? After all, you could have an even coarser grained detector that couldn't distinguish the -1 and +1 since everything between -5 and 5 looked the same to it. So there is no reason to split |0> into |+1>/√2+|-1>/√2 and yet split |-1> into |-.9>/2+|-1.1>/2 instead of |-.9>/√2+|-1.1>/√2. So any objection you have to doing it right, should have been a problem back at the beginning. – Timaeus Jan 07 '16 at 02:27
  • @user1247 Your why question is answered. – Timaeus Jan 07 '16 at 03:07
  • If I used the same procedure to split |0> into |+1>/2+|-1>/2 I would be perfectly fine because I would then just normalize the state and end up with |+1>/√2+|-1>/√2. So no, this was never a problem back at the beginning... but even if it was a problem back at the beginning I would still have to start an example some where to illustrate my confusion, so I find this kind of a red herring... – user1247 Jan 07 '16 at 15:39
  • @user1247 I did edit the question to include showing what principles you use and how you use them – Timaeus Jan 07 '16 at 15:48
  • OK, just read your edit, and it's very clear and very good, and I agree with and think I understand everything you wrote, but I don't think it really addresses my confusion. Sorry to be so difficult. Let me try one more time to explain exactly where my confusion lies. Please look back at the original picture I drew, on the top left, but for simplicity replace the bell curve shapes with square pulses over [-1.1,-0.9] and [0.9,1.1]. My understanding of what this represents is an integral over equal-weighted |x> over those two domains (with zero weight elsewhere). – user1247 Jan 07 '16 at 16:07
  • 1
    But in order for a Riemann-sum-based integral to make sense in this context, we should equally-well be able to replace this by 1/4|[-1.0,-1.1]>+1/4|[-0.9,-1.0]>+1/4|[0.9,1.0]>+1/4|[1.0,1.1]>, or by 1/8|[-1.05,-1.10]>+1/8|[-1.00,-1.05]>+1/8|[-0.95,-1.00]>+1/8|[-0.90,-0.95]>+1/8|[1.05,1.10]>+1/8|[1.00,1.05]>+1/8|[0.95,1.00]>+1/8|[0.90,0.95]>, and so on and so forth. But the confusion I have is that if you assume this then you get into trouble, since if you try to do a Riemann sum with unequal intervals, then Born rule gives you a different probability distribution, since... – user1247 Jan 07 '16 at 16:12
  • 1
    ...since the Born rule weighting is not linear for the different sized Riemann intervals. In your original answer you seemed to be saying that when you change the interval sizes in the Riemann sum you have to assume the Born non-linearity from the outset in order to get the final answer you want, and I pointed out that this seemed to beg the question, since it just pushes the question elsewhere, to: why is our integral representation of the state not linear the way a Riemann integral is (which I thought we could assume from linearity)? – user1247 Jan 07 '16 at 16:18
  • @user1247 I'm not trying to beg the question. I'm saying that the state is whatever the state is. And that different measurements devices will change that one fixed state in different ways. First make sure you fully know how to do the problem correctly (don't ever change the initial state). Then you'll probably be able to solve your own problem with less effort than trying to explain it to someone else. You need normalized states, and linear combinations and measurement device. I edited the answer again. – Timaeus Jan 07 '16 at 17:36
  • @user1247 Different sized intervals need different normalizations to be unit length. My newest edit includes a part at the top. You either have a mathematical error, at the top. Or a physics error (so learn to do it the right way). – Timaeus Jan 07 '16 at 18:04
  • "Just becasue you regroup a lknear combinations of six vectors into two groups of three, does not mean the coefficients in the linear combination change." -- but that's exactly what you are proposing I do, and exactly what I am NOT doing. You also say "I don't know why you bring up the word Riemann" as though I'm just throwing around random words, but I explained pretty carefully what I meant above. A Riemann sum is formed by breaking the domain into intervals and summing with weight going linearly as the interval size, but you are explicitly saying different ("break the pieces into 1/√2"). – user1247 Jan 07 '16 at 18:19
  • I think I'm realizing that my confusion comes down to the fact that distributions of psi(x) like shown in my picture represent an infinite sum of delta function states each with the same amplitude, and that funny stuff happens when you try to work with such states without mathematical rigor, as discussed eg here: http://physics.stackexchange.com/questions/155304/how-do-we-normalize-a-delta-function-position-space-wave-function – user1247 Jan 07 '16 at 19:25
  • Well the confusing thing is that if you have the state α1|−1.1⟩+α2|−0.9⟩+β|+1⟩ and then define the state α|-1> = α1|−1.1⟩+α2|−0.9⟩ then in order for the state to be normalized, α cannot equal α1 + α2. Maybe that just boils the problem down to something very simple. The problem is that this seems to directly contradict the Many Worlds viewpoint, since the "number of worlds" is not related linearly to the coursness of graining. Does that make any sense? – user1247 Jan 07 '16 at 19:40
  • @user1247 I don't think that is your confusion. It night be related to whatever you mean by bringing up Reimann (wavefunction are in general not Reimann integrable). You just aren't grouping correctly in terms of orthonormal vectors. Reread the entire new answer. – Timaeus Jan 07 '16 at 19:41
  • I re-read your edit. Quite the tour-de-force, thanks for the patience. As per my last comment, I think my confusion distills to the fact that Ψleft = 1√2Ψ1+1/√2Ψ2, and not 1/2Ψ1+1/2Ψ2. As I've tried to make clear, I realize that the latter leaves the state un-normalized, but intuitively the former makes no sense in the context of trying to associate the amplitude with "worlds," since here clearly the number of "worlds" in which the particle is found on the LHS cannot be represented by... – user1247 Jan 07 '16 at 20:05
  • ... by the amplitude, since it is not the simple addition of the amplitude for the particle to be found in the two halves of the LHS. In other words, even though in your example you insist that the √(5/4) never changes, in fact it does change by a factor of √2 when you re-label your axis basis states in terms of Ψ1 and Ψ2 and Ψright rather than Ψleft and Ψright. – user1247 Jan 07 '16 at 20:09
  • @user1247 Peter Shor's answer is clear, correct, and direct. The only reason I don't delete mine, is that it has more details about $\psi_1$ and $\Psi_1$ so you can tell why the coefficients are the way they are. Each capitalization or lack thereof is important. Please reread carefully so that you can ask about why I did anything I did. – Timaeus Jan 07 '16 at 20:10
  • My question boiled down to WHY Peter's terse statement make sense, in light of my other comments. He didn't address this at all, but rather repeated something I already know to be correct, so no, I think your attempt to answer my question is far more useful! Thanks! Again, please see my last two comments since you did not respond to them. – user1247 Jan 07 '16 at 20:24
  • @user1247 No, the function still has a value of $\sqrt{5/4}$ or 0 at each point. If you can't see that, you need to write the functions. All the lower case $\psi$ are themselves 0 or $\sqrt{5/4}$ at each point. The captial $\Psi_1,$ $\Psi_2$ and $\Psi_{\text{left}}$ are actually larger which is why we have to multiply by a smaller linear combination coefficient. Note that Peter Shor correctly told you in his comment that you have to square and then add/integrate. It is just the Pythagorean theorem, the squared norm of the vector sum is the sum of the squared norms of the orthogonal vectors – Timaeus Jan 07 '16 at 20:31
  • I drew an illustration and edited my question. The new picture is at the bottom of the question. It's hard for me to see how there can be any confusion about the fact that the area under the curve is changed. I am aware that you have to square and then add/integrate, and don't see where I may have made that mistake. Again, I think Peter doesn't understand what my confusion is, if he thinks it stems from me not knowing the difference between amplitude and probability being the norm squared amplitude. – user1247 Jan 07 '16 at 20:54
  • (note: in my browser it's really really difficult to see the difference between your bold/capital and uncapitalized psis, and so maybe that's the problem, but it doesn't work out for me when I do the substitutions, it doesn't come out normalized) – user1247 Jan 07 '16 at 21:00
  • @user1247 I changed the lower case to $\phi$ so they should look totally different. – Timaeus Jan 08 '16 at 01:30
  • Thanks, this time I was able to follow your math through, and I agree that your final function psi has amplitude root 5/4 in the relevant domain. But my point wasn't that once you unravel everything back in terms of your original basis states that anything changed, my point was that when you move between course and fine-grained basis states the amplitude changes (non-linearly). Again this can be seen directly in the second picture I added, or just by looking at the final form of your wave function in terms of capital psis. – user1247 Jan 08 '16 at 15:58
  • Here is another stab at understanding my core conceptual confusion. If we use |x> as our basis states, they are the eigenvectors of the position operator. If we use phi as our basis states, they are eigenvectors of some kind of pseudo-position operator with some granularity, and if we use Psi as our basis states they are eigenvectors of a pseudo-position operator with lower granularity. Finding the relative normalizations here would seem a lot less ad-hoc if instead of just rescaling things using your unit norm method to force everything to stay normalized, if we explicitly... – user1247 Jan 08 '16 at 16:08
  • ...explicitly derived the relationship between the fine and course-grained basis states by defining the relevant operators and finding their eigenvalue spectra in terms of each other. Does that make sense? – user1247 Jan 08 '16 at 16:10
  • @user1247 You could just do the projection onto the eigenspaces of the operator corresponding to the measurement. All of this writing in terms of a basis is just one way (of many) to compute the ratio of these squared norms. And I still don't think you understood what I wrote. The only difference between a $\Psi$ and a $\phi$ is the first is a unit vector and the other is not. The vector $\phi_{\text{left}}$ is an eigenvector of the coarser grained operator and $\phi_1$ and $\phi_2$ are eigenvectors of the finer grained operator. $\phi_{\text{left}}=\phi_1+\phi_2.$ But not so for the $\Psi$s. – Timaeus Jan 08 '16 at 18:54
  • Maybe see Peter's most recent comments under his post, because I think he understands me. Also, it would be nice if you commented on the picture I added, where I use the correct normalization for the states but still end up with a clearly different area under the curve. I don't think it's clear that phi_left is strictly speaking an eigenvector of the coarser grained operator because we haven't established that the eigenvalue of the courser grained operator will be 1/2 the sum of the eigenvalues of the finer grained operator. – user1247 Jan 08 '16 at 20:17
  • @user1247 In your comments to Peter you clearly indicate that you think the the coarse detector changes it to the position eigenstate $|-1\rangle$ 50% of the time. But that is false. It makes me regret deleting so much in the edits. But you clearly don't understand my first paragraph. The rest is just math. – Timaeus Jan 08 '16 at 20:41
  • This is a very confusing statement, can you elaborate? For the course detector with eigenstates |+1> and |-1> the probabilities are trivially 50% each, so I don't understand what you mean. In my comments to Peter I was pretty explicit in agreeing with him on the math. I only pressed a conceptual issue, so I'm not sure what you are referring to there either. – user1247 Jan 09 '16 at 04:09
  • @user1247 You are oh so wrong, Imagine you had a matrix and someone told you two of the columns. They haven't told you the matrix. So sure, $|-1\rangle$ and $+1\rangle$ can be eigenvectors, but that is only saying where two vectors are sent. That's just specifying two columns. An operator has to tell you where every vector goes. If you decide that the operator sends every other vector to zero then it hardly ever goes off. You seriously need to write down once and for all (in any basis you want) what the initial state and the operator are. And then stick to it. Seriously. – Timaeus Jan 09 '16 at 14:34
  • If you want to help me, you need to be more clear, because I know for a fact (from every resource on the planet) that the state 1/sqrt(2)|+1>+1/sqrt(2)|-1> has via the Born rule a 50% chance of reducing to +1 or -1. This is not controversial, so if you are trying to say something different, please make that clear. – user1247 Jan 09 '16 at 16:38
  • This is in virtually every textbook discussing Stern-Gerlch apparatus with a charged particle being deflect left or right with 50% probability of pointer state being |-1> or |+1> – user1247 Jan 09 '16 at 16:40
  • @user1247 If you actually literally refuse to pick a single state and pick two operators, then I literally cannot answer your question about how to compute what happens. Because until you actually pick a state and two operators you don't have a question that is precise enough to have an answer. It is clearly not helpful for you to be vague about your state or to be vague about your operator, and if you cut it out, then you can get an answer. You cannot compute probabilities until you specify a state and an operator. Please clearly pick a state and two operators and stick to it. – Timaeus Jan 09 '16 at 17:01
  • According to Peter's comments I cannot do that, at least without some help. Also, I did explicitly ask you for help in that regard, here I quote from myself: "If we use |x> as our basis states, they are the eigenvectors of the position operator. If we use phi as our basis states, they are eigenvectors of some kind of pseudo-position operator with some granularity, and if we use Psi as our basis states they are eigenvectors of a pseudo-position operator with lower granularity." Now I'm not sure how all of this works, and would value your guidance, but you never really helped me do that. – user1247 Jan 10 '16 at 17:46
  • @user1247 When you said that it was just plain wrong and I told you to go back and reread. The only difference between $\phi$ and $\Psi$ is that the latter is a unit norm version of the former. If you think otherwise then you misunderstood everything. And I don't think you need help writing a state and some operators. If you had four triangels then the first operator could send the left two to -1 times themselves and send the right two to themselves, all four are eigenvectors. The less coarse operator could send the four triangels to -2 -1 +1 and +2 times themselves respectively (for example). – Timaeus Jan 10 '16 at 17:53
  • I don't know what "wrong" you are referring to. I realize that the difference between phi and Psi is that the latter is a unit norm of the other and have no idea why you would think I do not understand that. You did say that I was wrong that the probability of measuring +1 in a state 1/sqrt(2)|+1>+1/sqrt(2)|-1> is 50%, which is perplexing and indicates you are not being clear and we are talking past each other. Peter made it clear that you are wrong to think we can form the operators you are describing, maybe read his comment and respond. Also, you never responded at all to the picture I added – user1247 Jan 10 '16 at 20:07
  • @user1247 Whether you use unit vectors or non unit vectors has zero effect on what operators they are eigenvectors to. And saying otherwise over and over is eventually just being wrong on purpose. Hopefully you aren't going to seriously claim that rescaling a vector changes what matrix it is eigen to. I've removed any names for vectors that aren't unit length so you won't think that makes them change what they are eigen to. And Peter never claimed you couldn't make the operators. You just were choosing not to make them. So I made them. And you do know about eigenspaces. I edited for clarity. – Timaeus Jan 11 '16 at 00:38
  • Your new edit is truly wonderful and goes a long way. I really do want to thank you for how much work you've put in, despite that there has been some difficulty communicating. I think the biggest thing that helped was going from your first to second equation in the H atom. I had never heard that the Born rule includes the provision "if the experimental outcome doesn't distinguish between states A and B, then the probability for that outcome is the sum of the corresponding probabilities for A and B." If that's really part of the Born rule then I agree everything else follows. One major thing... – user1247 Jan 11 '16 at 02:33
  • ...thing that is missing, however, is that you do not do what I was asking, that is, explicitly show that if you write the eigenvectors of LR in terms of the eigenvectors of ABC, that you end up with the required 1/sqrt(2) proportionality factor that gave rise to all of my conceptual confusion (and which made me worry the factor was added in an "ad-hoc" manner in order to circularly insure that you up with the expected probability). Looking at the definition of your eigenvectors, it's not clear to me how one would derive that relationship, since the eigenvectors of both... – user1247 Jan 11 '16 at 02:36
  • ...both LR and ABC have the same amplitude when x is in the respective domain. I hope that you're willing to answer that confusion, but as a sign of good faith I'm awarding you the accepted answer. Thanks! – user1247 Jan 11 '16 at 02:37
  • @user1247 I isted nine steps in the Born Rule, and didn't mention your quote, which isn't even something I can parse. The projections I mention are the orthogonal projections and they are orthogonal in the standard inner product for the Hilbert Space. I didn't mean anything to be unclear. As for the 1/2 and the 1/√2 you know the function needs to be √(5/4) where it isn't zero. And you know the magnitude of the vectors in the ABC eigenspaces so you can figure out how to scale to get a match. But that isn't the only method. You're just writing one vector in terms of orthonormal basis ... – Timaeus Jan 11 '16 at 02:52
  • If you can't parse it then I better try to make sure you understand, because it seems to be a pretty big assumption. When you write "Then afterwards you end up with |100⟩⊗|13.6eV⟩ 50% of the time," how do you justify that statement using any of your nine steps? – user1247 Jan 11 '16 at 02:56
  • @user1247 There exists an orthonormal basis of eigenvectors for any operator. So let O have eigenvectors e1,e2, e3, ... And let v be a vector. Then v=<e1,v>e1+<e2,v>e2+<e3,v>e3+... Which you can verify by taking the inner product of both sides by ei. And so once you inner product your wave with the two or three orthonormal vectors and get coefficients whose squares add up to one you know you are done because the rest will have to give you zero. – Timaeus Jan 11 '16 at 02:57
  • @user1247 I projected the vector into the eigenspace (and got |100⟩/√2) computed the squared norm of the projection (I got 1/2), computed the squared norm of the original (I got 1), took the ratio of the smaller one (1/2) over the larger one (1) and got (1/2)/(1)=1/2. You should do those steps too. – Timaeus Jan 11 '16 at 03:01
  • I did do those steps, but you are "cheating" by using the fact that you know the probabilities have to add up to one, so you used the Born rule for the non-composite piece (|100>) in order to infer the probability for the composite piece (|211>+|210>). I guess maybe that is OK (and maybe you can always do that in more general cases), but that's what I meant by it not being in your list. – user1247 Jan 11 '16 at 03:09
  • Wait, actually, it doesn't work in the general case, for example if you had 5 states, and wanted to compare the probability of getting a 2-state degenerate subset to a 3-state degenerate subset... you would need to add an additional item to your list to be able to treat such cases in general, no? – user1247 Jan 11 '16 at 03:11
  • @user1247 I didn't do that at all. I'm just writing a vector in terms of orthonormal vectors so that it is easier to compute projections into eigenspaces generated by those orthonormal vectors. – Timaeus Jan 11 '16 at 03:11
  • @user1247 I can not figure out what case you are talking about. To be an observable requires that you have an orthonormal basis made entirely of eigenvectors. Sometimes many many such sets.. You can use any of them to do projections easily. But you can do projections without them as well. Do you have trouble finding eigenvectors? Trouble finding orthonormal ones? Trouble using them to do a projection? Trouble finding norms? Make sure you know how to do all nine steps. – Timaeus Jan 11 '16 at 03:15
  • Oh, you're right, I mis-read what you were doing. – user1247 Jan 11 '16 at 03:47
2

The problem is that the wave function should not be:

$$\frac{1}{\sqrt{2}}|x = +1\rangle + \frac{1}{\sqrt{2}} \left( \frac{1}{2}|x = -1.01 \rangle + \frac{1}{2} |x = -.99 \rangle \right) $$

but $$\frac{1}{\sqrt{2}}|x = +1\rangle + \frac{1}{\sqrt{2}} \left( \frac{1}{\sqrt{2}}|x = -1.01 \rangle + \frac{1}{\sqrt{2}} |x = -.99 \rangle \right). $$

Peter Shor
  • 11,274
  • I understand that that is what it should be, but I seem to be having trouble translating between the continuous case shown in my picture and the discrete cases like above. What I want to do is associate the area under the curve with the amplitude for a discrete state, but that doesn't seem to work (in the middle panel of my example I see that there is half the area under the curve for each side of the amplitude lump on the LHS, and so I want to say that the amplitude for a discrete state representing each half is just half, and it's strange this transforms into 1/√2). – user1247 Jan 07 '16 at 19:31
  • 3
    For the continuous case, you integrate the absolute value of the square of the amplitude to get the probability, not the amplitude. The integral of the area under the square of the curve is the probability, not the integral of the area under the curve, which is what your transformations are assuming. – Peter Shor Jan 07 '16 at 19:48
  • Maybe one way to see why your calculation is wrong is to imagine the amplitude around -1 made up of two components, one positive and one negative. In the measurement with coarse resolution, your recipe would say that the chance of measuring -1 is zero, because the integral is 0. But in the measurement with fine resolution, you have a reasonable chance of measuring both -.99 and -1.01. Do you think quantum measurements work like that? – Peter Shor Jan 08 '16 at 15:07
  • That is a really good example Peter! I think conceptually I wouldn't actually want to say that the chance of measuring -1 is zero, because I would be working with a "many worlds" ontology where I would emphasize that both the -0.99 and -1.01 states have equal normed amplitude despite having both positive and negative phase component. So the real issue is why, when we lump both states together, is the normed amplitude not the linear addition of the normed amplitudes. Obviously that's just not how QM works, but it's confusing trying to understand given a "wave function is real" ontology. – user1247 Jan 08 '16 at 15:27
  • Note that if it did work the way I would want it to work, there would still be quantumy interference effects, but if you moved between fine and course grainings the integral over normed amplitude in any given region would stay constant. – user1247 Jan 08 '16 at 15:29
  • Is there any way of deriving the correct wave function on the second line of your answer by finding the eigenvalue spectra of the course and fine-grained position operators and then writing them in terms of each other? Rather than the ad-hoc method of writing something down so that the state remains normalized? – user1247 Jan 08 '16 at 16:12
  • It's not ad hoc. You just have to realize that integrating amplitudes to get the coarse-grained amplitude is not the right way to do things. You seem to be trying to do this using the simplest form of quantum measurement (probably the only one you've been taught) where everything gets projected onto orthogonal one-dimensional vectors. This doesn't work ... you need to understand more general quantum measurements. I don't believe there is any such thing as the "eigenvalue spectrum of the coarse-grained position operator" in the sense that you want. – Peter Shor Jan 08 '16 at 16:15
  • Presumably there is an operator with eigenvectors |x=+1> and |x=-1> corresponding to the two possible experimental pointer states, and a different operator with eigenvectors |+1> and |x=-0.99> and |x=-1.01> corresponding to a pointer states for an improved version of the detector. That's not right? Is there a good resource you can recommend for the "more general quantum measurements"? I do have a number of standard QM texts from back when I took QM classes. – user1247 Jan 08 '16 at 17:18
  • No, there's not. They teach this simplified version of measurement in QM classes, and it's completely inadequate when you start asking questions like this. The book Quantum Theory: Concepts and Methods by Asher Peres is quite good for these more general types of measurements. – Peter Shor Jan 08 '16 at 18:51
  • You need to replace "eigenvectors" with "eigenspaces" in your comment above to get something that's close to correct. (It's a measurement operator, but it's not the most general form of measurement operator.) – Peter Shor Jan 08 '16 at 18:58